Prove continuity on a function at every irrational point and discontinuity at every rational point.

The name of the pictureThe name of the pictureThe name of the pictureClash Royale CLAN TAG#URR8PPP











up vote
12
down vote

favorite
10












Consider the function:



$f(x)= begincases
1/n quad &textif $x= m/n$ in simplest form \
0 quad &textif $x in mathbbRsetminusmathbbQ$
endcases $



Prove that the function is continuous at every irrational point and also that the function is not continuous at every rational point. Also, we can say that the function is continuous at some point $k$ if $displaystylelim_x to k f(x)=f(k)$.



I was thinking of doing an epsilon delta proof backwards using the fact that $mathbbQ$ is dense in $mathbbR$ for rational points and irrational points. Any ways on how to expand on this are welcome.







share|cite|improve this question















  • 2




    The way to do this is to show that in arbitrarily small neighborhoods of an irrational point $x$, rationals approximating $x$ must have large denominators in reduced form.
    – Seth
    Oct 4 '12 at 12:11







  • 1




    Hint: $|frac ab-frac xy|=|fracay-bxby|ge frac1by$ if $frac abnefrac xy$
    – Hagen von Eitzen
    Oct 4 '12 at 12:31















up vote
12
down vote

favorite
10












Consider the function:



$f(x)= begincases
1/n quad &textif $x= m/n$ in simplest form \
0 quad &textif $x in mathbbRsetminusmathbbQ$
endcases $



Prove that the function is continuous at every irrational point and also that the function is not continuous at every rational point. Also, we can say that the function is continuous at some point $k$ if $displaystylelim_x to k f(x)=f(k)$.



I was thinking of doing an epsilon delta proof backwards using the fact that $mathbbQ$ is dense in $mathbbR$ for rational points and irrational points. Any ways on how to expand on this are welcome.







share|cite|improve this question















  • 2




    The way to do this is to show that in arbitrarily small neighborhoods of an irrational point $x$, rationals approximating $x$ must have large denominators in reduced form.
    – Seth
    Oct 4 '12 at 12:11







  • 1




    Hint: $|frac ab-frac xy|=|fracay-bxby|ge frac1by$ if $frac abnefrac xy$
    – Hagen von Eitzen
    Oct 4 '12 at 12:31













up vote
12
down vote

favorite
10









up vote
12
down vote

favorite
10






10





Consider the function:



$f(x)= begincases
1/n quad &textif $x= m/n$ in simplest form \
0 quad &textif $x in mathbbRsetminusmathbbQ$
endcases $



Prove that the function is continuous at every irrational point and also that the function is not continuous at every rational point. Also, we can say that the function is continuous at some point $k$ if $displaystylelim_x to k f(x)=f(k)$.



I was thinking of doing an epsilon delta proof backwards using the fact that $mathbbQ$ is dense in $mathbbR$ for rational points and irrational points. Any ways on how to expand on this are welcome.







share|cite|improve this question











Consider the function:



$f(x)= begincases
1/n quad &textif $x= m/n$ in simplest form \
0 quad &textif $x in mathbbRsetminusmathbbQ$
endcases $



Prove that the function is continuous at every irrational point and also that the function is not continuous at every rational point. Also, we can say that the function is continuous at some point $k$ if $displaystylelim_x to k f(x)=f(k)$.



I was thinking of doing an epsilon delta proof backwards using the fact that $mathbbQ$ is dense in $mathbbR$ for rational points and irrational points. Any ways on how to expand on this are welcome.









share|cite|improve this question










share|cite|improve this question




share|cite|improve this question









asked Oct 4 '12 at 11:54









Casquibaldo

3721720




3721720







  • 2




    The way to do this is to show that in arbitrarily small neighborhoods of an irrational point $x$, rationals approximating $x$ must have large denominators in reduced form.
    – Seth
    Oct 4 '12 at 12:11







  • 1




    Hint: $|frac ab-frac xy|=|fracay-bxby|ge frac1by$ if $frac abnefrac xy$
    – Hagen von Eitzen
    Oct 4 '12 at 12:31













  • 2




    The way to do this is to show that in arbitrarily small neighborhoods of an irrational point $x$, rationals approximating $x$ must have large denominators in reduced form.
    – Seth
    Oct 4 '12 at 12:11







  • 1




    Hint: $|frac ab-frac xy|=|fracay-bxby|ge frac1by$ if $frac abnefrac xy$
    – Hagen von Eitzen
    Oct 4 '12 at 12:31








2




2




The way to do this is to show that in arbitrarily small neighborhoods of an irrational point $x$, rationals approximating $x$ must have large denominators in reduced form.
– Seth
Oct 4 '12 at 12:11





The way to do this is to show that in arbitrarily small neighborhoods of an irrational point $x$, rationals approximating $x$ must have large denominators in reduced form.
– Seth
Oct 4 '12 at 12:11





1




1




Hint: $|frac ab-frac xy|=|fracay-bxby|ge frac1by$ if $frac abnefrac xy$
– Hagen von Eitzen
Oct 4 '12 at 12:31





Hint: $|frac ab-frac xy|=|fracay-bxby|ge frac1by$ if $frac abnefrac xy$
– Hagen von Eitzen
Oct 4 '12 at 12:31











2 Answers
2






active

oldest

votes

















up vote
7
down vote













You cannot do this using simple rules about limits, like de l?Hôpital's rule or the continuity of "analytic expressions", etc. You have to go back to the actual definition of continuity when solving this problem. Here are a few hints:



In the first place you want to prove different things about a given $xinmathbb R$, depending on whether $x$ is rational or irrational.



If $x$ is rational then $f(x)$ is a certain positive number. It is claimed that $f$ is not continuous at $x$. Assume to the contrary that $f$ is continuous at $x$. Then there is a neighborhood $U:=U_delta(x)$ such that $f(t)>f(x)over2$ for all $tin U$. Hm?



If $x$ is irrational then $f(x)=0$. It is claimed that $f$ is continuous at $x$. This means that for any $epsilon>0$ we should be able to produce a neighborhood $U:=U_delta(x)$ such that $f(t)<epsilon$ for all $tin U$. Which points $tinmathbb R$ could violate the condition $f(t)<epsilon $?






share|cite|improve this answer





















  • Are they going to be the rationals?
    – Casquibaldo
    Oct 4 '12 at 13:27










  • @Casquibaldo: If you mean by "they" the $t$ in the last line of my answer: Yes, but only selected rationals.
    – Christian Blatter
    Oct 4 '12 at 13:31

















up vote
5
down vote













This function is known as Thomae's function and is an excellent example of a function that is continuous at uncountably infinite number of points while discontinuous at countably infinite number of points. Note that the reverse result is not possible, i.e. one cannot have a function that is discontinuous on an uncountably infinite dense set and continuous on a countably infinite dense set.



As for the proof , the discontinuity part follows from the fact that irrationals are dense in $ R $ as you rightly thought. The continuity part is a little bit more involved and it would help to employ the Archimedean principle in tandem with the $ epsilon$-$delta $ definition.



Crudely, look at $ x=sqrt3 $ , see that using the non-terminating decimal expansion i.e. $ sqrt 3=1.732148ldots $ , one can forge a sequence of rationals $ 1 , 17/10 ,173/100 ,ldots $
that converges to $ x $, while at the same time your $ f(x_n) $ is $ 1/10,1/100,1/1000,ldots $ which converges to $ 0 $ which is nothing but $ f(x) $. This is merely an illustration of the continuity and not a rigorous proof.






share|cite|improve this answer























    Your Answer




    StackExchange.ifUsing("editor", function ()
    return StackExchange.using("mathjaxEditing", function ()
    StackExchange.MarkdownEditor.creationCallbacks.add(function (editor, postfix)
    StackExchange.mathjaxEditing.prepareWmdForMathJax(editor, postfix, [["$", "$"], ["\\(","\\)"]]);
    );
    );
    , "mathjax-editing");

    StackExchange.ready(function()
    var channelOptions =
    tags: "".split(" "),
    id: "69"
    ;
    initTagRenderer("".split(" "), "".split(" "), channelOptions);

    StackExchange.using("externalEditor", function()
    // Have to fire editor after snippets, if snippets enabled
    if (StackExchange.settings.snippets.snippetsEnabled)
    StackExchange.using("snippets", function()
    createEditor();
    );

    else
    createEditor();

    );

    function createEditor()
    StackExchange.prepareEditor(
    heartbeatType: 'answer',
    convertImagesToLinks: true,
    noModals: false,
    showLowRepImageUploadWarning: true,
    reputationToPostImages: 10,
    bindNavPrevention: true,
    postfix: "",
    noCode: true, onDemand: true,
    discardSelector: ".discard-answer"
    ,immediatelyShowMarkdownHelp:true
    );



    );








     

    draft saved


    draft discarded


















    StackExchange.ready(
    function ()
    StackExchange.openid.initPostLogin('.new-post-login', 'https%3a%2f%2fmath.stackexchange.com%2fquestions%2f207118%2fprove-continuity-on-a-function-at-every-irrational-point-and-discontinuity-at-ev%23new-answer', 'question_page');

    );

    Post as a guest






























    2 Answers
    2






    active

    oldest

    votes








    2 Answers
    2






    active

    oldest

    votes









    active

    oldest

    votes






    active

    oldest

    votes








    up vote
    7
    down vote













    You cannot do this using simple rules about limits, like de l?Hôpital's rule or the continuity of "analytic expressions", etc. You have to go back to the actual definition of continuity when solving this problem. Here are a few hints:



    In the first place you want to prove different things about a given $xinmathbb R$, depending on whether $x$ is rational or irrational.



    If $x$ is rational then $f(x)$ is a certain positive number. It is claimed that $f$ is not continuous at $x$. Assume to the contrary that $f$ is continuous at $x$. Then there is a neighborhood $U:=U_delta(x)$ such that $f(t)>f(x)over2$ for all $tin U$. Hm?



    If $x$ is irrational then $f(x)=0$. It is claimed that $f$ is continuous at $x$. This means that for any $epsilon>0$ we should be able to produce a neighborhood $U:=U_delta(x)$ such that $f(t)<epsilon$ for all $tin U$. Which points $tinmathbb R$ could violate the condition $f(t)<epsilon $?






    share|cite|improve this answer





















    • Are they going to be the rationals?
      – Casquibaldo
      Oct 4 '12 at 13:27










    • @Casquibaldo: If you mean by "they" the $t$ in the last line of my answer: Yes, but only selected rationals.
      – Christian Blatter
      Oct 4 '12 at 13:31














    up vote
    7
    down vote













    You cannot do this using simple rules about limits, like de l?Hôpital's rule or the continuity of "analytic expressions", etc. You have to go back to the actual definition of continuity when solving this problem. Here are a few hints:



    In the first place you want to prove different things about a given $xinmathbb R$, depending on whether $x$ is rational or irrational.



    If $x$ is rational then $f(x)$ is a certain positive number. It is claimed that $f$ is not continuous at $x$. Assume to the contrary that $f$ is continuous at $x$. Then there is a neighborhood $U:=U_delta(x)$ such that $f(t)>f(x)over2$ for all $tin U$. Hm?



    If $x$ is irrational then $f(x)=0$. It is claimed that $f$ is continuous at $x$. This means that for any $epsilon>0$ we should be able to produce a neighborhood $U:=U_delta(x)$ such that $f(t)<epsilon$ for all $tin U$. Which points $tinmathbb R$ could violate the condition $f(t)<epsilon $?






    share|cite|improve this answer





















    • Are they going to be the rationals?
      – Casquibaldo
      Oct 4 '12 at 13:27










    • @Casquibaldo: If you mean by "they" the $t$ in the last line of my answer: Yes, but only selected rationals.
      – Christian Blatter
      Oct 4 '12 at 13:31












    up vote
    7
    down vote










    up vote
    7
    down vote









    You cannot do this using simple rules about limits, like de l?Hôpital's rule or the continuity of "analytic expressions", etc. You have to go back to the actual definition of continuity when solving this problem. Here are a few hints:



    In the first place you want to prove different things about a given $xinmathbb R$, depending on whether $x$ is rational or irrational.



    If $x$ is rational then $f(x)$ is a certain positive number. It is claimed that $f$ is not continuous at $x$. Assume to the contrary that $f$ is continuous at $x$. Then there is a neighborhood $U:=U_delta(x)$ such that $f(t)>f(x)over2$ for all $tin U$. Hm?



    If $x$ is irrational then $f(x)=0$. It is claimed that $f$ is continuous at $x$. This means that for any $epsilon>0$ we should be able to produce a neighborhood $U:=U_delta(x)$ such that $f(t)<epsilon$ for all $tin U$. Which points $tinmathbb R$ could violate the condition $f(t)<epsilon $?






    share|cite|improve this answer













    You cannot do this using simple rules about limits, like de l?Hôpital's rule or the continuity of "analytic expressions", etc. You have to go back to the actual definition of continuity when solving this problem. Here are a few hints:



    In the first place you want to prove different things about a given $xinmathbb R$, depending on whether $x$ is rational or irrational.



    If $x$ is rational then $f(x)$ is a certain positive number. It is claimed that $f$ is not continuous at $x$. Assume to the contrary that $f$ is continuous at $x$. Then there is a neighborhood $U:=U_delta(x)$ such that $f(t)>f(x)over2$ for all $tin U$. Hm?



    If $x$ is irrational then $f(x)=0$. It is claimed that $f$ is continuous at $x$. This means that for any $epsilon>0$ we should be able to produce a neighborhood $U:=U_delta(x)$ such that $f(t)<epsilon$ for all $tin U$. Which points $tinmathbb R$ could violate the condition $f(t)<epsilon $?







    share|cite|improve this answer













    share|cite|improve this answer



    share|cite|improve this answer











    answered Oct 4 '12 at 12:52









    Christian Blatter

    164k7107306




    164k7107306











    • Are they going to be the rationals?
      – Casquibaldo
      Oct 4 '12 at 13:27










    • @Casquibaldo: If you mean by "they" the $t$ in the last line of my answer: Yes, but only selected rationals.
      – Christian Blatter
      Oct 4 '12 at 13:31
















    • Are they going to be the rationals?
      – Casquibaldo
      Oct 4 '12 at 13:27










    • @Casquibaldo: If you mean by "they" the $t$ in the last line of my answer: Yes, but only selected rationals.
      – Christian Blatter
      Oct 4 '12 at 13:31















    Are they going to be the rationals?
    – Casquibaldo
    Oct 4 '12 at 13:27




    Are they going to be the rationals?
    – Casquibaldo
    Oct 4 '12 at 13:27












    @Casquibaldo: If you mean by "they" the $t$ in the last line of my answer: Yes, but only selected rationals.
    – Christian Blatter
    Oct 4 '12 at 13:31




    @Casquibaldo: If you mean by "they" the $t$ in the last line of my answer: Yes, but only selected rationals.
    – Christian Blatter
    Oct 4 '12 at 13:31










    up vote
    5
    down vote













    This function is known as Thomae's function and is an excellent example of a function that is continuous at uncountably infinite number of points while discontinuous at countably infinite number of points. Note that the reverse result is not possible, i.e. one cannot have a function that is discontinuous on an uncountably infinite dense set and continuous on a countably infinite dense set.



    As for the proof , the discontinuity part follows from the fact that irrationals are dense in $ R $ as you rightly thought. The continuity part is a little bit more involved and it would help to employ the Archimedean principle in tandem with the $ epsilon$-$delta $ definition.



    Crudely, look at $ x=sqrt3 $ , see that using the non-terminating decimal expansion i.e. $ sqrt 3=1.732148ldots $ , one can forge a sequence of rationals $ 1 , 17/10 ,173/100 ,ldots $
    that converges to $ x $, while at the same time your $ f(x_n) $ is $ 1/10,1/100,1/1000,ldots $ which converges to $ 0 $ which is nothing but $ f(x) $. This is merely an illustration of the continuity and not a rigorous proof.






    share|cite|improve this answer



























      up vote
      5
      down vote













      This function is known as Thomae's function and is an excellent example of a function that is continuous at uncountably infinite number of points while discontinuous at countably infinite number of points. Note that the reverse result is not possible, i.e. one cannot have a function that is discontinuous on an uncountably infinite dense set and continuous on a countably infinite dense set.



      As for the proof , the discontinuity part follows from the fact that irrationals are dense in $ R $ as you rightly thought. The continuity part is a little bit more involved and it would help to employ the Archimedean principle in tandem with the $ epsilon$-$delta $ definition.



      Crudely, look at $ x=sqrt3 $ , see that using the non-terminating decimal expansion i.e. $ sqrt 3=1.732148ldots $ , one can forge a sequence of rationals $ 1 , 17/10 ,173/100 ,ldots $
      that converges to $ x $, while at the same time your $ f(x_n) $ is $ 1/10,1/100,1/1000,ldots $ which converges to $ 0 $ which is nothing but $ f(x) $. This is merely an illustration of the continuity and not a rigorous proof.






      share|cite|improve this answer

























        up vote
        5
        down vote










        up vote
        5
        down vote









        This function is known as Thomae's function and is an excellent example of a function that is continuous at uncountably infinite number of points while discontinuous at countably infinite number of points. Note that the reverse result is not possible, i.e. one cannot have a function that is discontinuous on an uncountably infinite dense set and continuous on a countably infinite dense set.



        As for the proof , the discontinuity part follows from the fact that irrationals are dense in $ R $ as you rightly thought. The continuity part is a little bit more involved and it would help to employ the Archimedean principle in tandem with the $ epsilon$-$delta $ definition.



        Crudely, look at $ x=sqrt3 $ , see that using the non-terminating decimal expansion i.e. $ sqrt 3=1.732148ldots $ , one can forge a sequence of rationals $ 1 , 17/10 ,173/100 ,ldots $
        that converges to $ x $, while at the same time your $ f(x_n) $ is $ 1/10,1/100,1/1000,ldots $ which converges to $ 0 $ which is nothing but $ f(x) $. This is merely an illustration of the continuity and not a rigorous proof.






        share|cite|improve this answer















        This function is known as Thomae's function and is an excellent example of a function that is continuous at uncountably infinite number of points while discontinuous at countably infinite number of points. Note that the reverse result is not possible, i.e. one cannot have a function that is discontinuous on an uncountably infinite dense set and continuous on a countably infinite dense set.



        As for the proof , the discontinuity part follows from the fact that irrationals are dense in $ R $ as you rightly thought. The continuity part is a little bit more involved and it would help to employ the Archimedean principle in tandem with the $ epsilon$-$delta $ definition.



        Crudely, look at $ x=sqrt3 $ , see that using the non-terminating decimal expansion i.e. $ sqrt 3=1.732148ldots $ , one can forge a sequence of rationals $ 1 , 17/10 ,173/100 ,ldots $
        that converges to $ x $, while at the same time your $ f(x_n) $ is $ 1/10,1/100,1/1000,ldots $ which converges to $ 0 $ which is nothing but $ f(x) $. This is merely an illustration of the continuity and not a rigorous proof.







        share|cite|improve this answer















        share|cite|improve this answer



        share|cite|improve this answer








        edited Oct 30 '16 at 5:43









        Michael Hardy

        204k23186462




        204k23186462











        answered Oct 4 '12 at 16:12









        Vishesh

        1,4661126




        1,4661126






















             

            draft saved


            draft discarded


























             


            draft saved


            draft discarded














            StackExchange.ready(
            function ()
            StackExchange.openid.initPostLogin('.new-post-login', 'https%3a%2f%2fmath.stackexchange.com%2fquestions%2f207118%2fprove-continuity-on-a-function-at-every-irrational-point-and-discontinuity-at-ev%23new-answer', 'question_page');

            );

            Post as a guest













































































            Comments

            Popular posts from this blog

            Color the edges and diagonals of a regular polygon

            Relationship between determinant of matrix and determinant of adjoint?

            What is the equation of a 3D cone with generalised tilt?